LSAT and Law School Admissions Forum

Get expert LSAT preparation and law school admissions advice from PowerScore Test Preparation.

User avatar
 Dave Killoran
PowerScore Staff
  • PowerScore Staff
  • Posts: 5852
  • Joined: Mar 25, 2011
|
#44246
Complete Question Explanation
(The complete setup for this game can be found here: lsat/viewtopic.php?t=8601)

The correct answer choice is (A)

To attack this question, we must find the crew list that cannot complete all of the tasks listed. It is important to note that each of these lists represents a complete crew (not four out of five members). Thus it is valuable to focus on the abilities of the crew members listed. The correct answer is answer choice (A), because between H, I, K, and M, the crew has no one to do the taping or the wallboarding, so this could not be an acceptable crew.

Get the most out of your LSAT Prep Plus subscription.

Analyze and track your performance with our Testing and Analytics Package.